Đến nội dung

Hình ảnh

BĐT AM-GM


  • Please log in to reply
Chủ đề này có 339 trả lời

#321
Sherlock Homes

Sherlock Homes

    Binh nhì

  • Thành viên
  • 13 Bài viết

Cho a,b,c là độ dài các cạnh của một tam giác. Chứng minh rằng 


Bài viết đã được chỉnh sửa nội dung bởi Sherlock Homes: 02-09-2016 - 12:10


#322
Sherlock Homes

Sherlock Homes

    Binh nhì

  • Thành viên
  • 13 Bài viết

Thách bạn nào làm được, làm được cho 1 Like :like  

Cho $ \left\{\begin{matrix} a,b,c>0 & \\ a+b+c=1& \end{matrix}\right.$

 

Tìm Min của S=$\frac{a^2+b}{b+c}+\frac{b^2+c}{c+a}+\frac{c^2+a}{a+b}$


Bài viết đã được chỉnh sửa nội dung bởi Sherlock Homes: 09-09-2016 - 21:47

Impossible is nothing --- ADDIDAS ---

     

                                                                                BELIVEVE THAT --- Roman Reigns ---


#323
Thutrau

Thutrau

    Binh nhì

  • Thành viên mới
  • 13 Bài viết

Anh Quang à ! Anh có lòng vậy thì anh chém hộ em luôn cái bài này nha . Lớp 8 thôi ? laugh.gif
Câu 1 :
Cho $a,b,c > 0 ; a+b+c \leq 1$ . Chứng minh rằng :
$\frac{1}{a^{2}+2bc} + \frac{1}{b^{2}+2ca} + \frac{1}{c^{2}+ 2ab}$ $\frac{1}{a^{2}+2bc} + \frac{1}{b^{2}+2ca} + \frac{1}{c^{2}+ 2ab}\geq 9$
Câu 2 :
Cho $a;b;c \geq 0 ; a+b+c =1$
Chứng minh rằng :
a, $\sqrt{a+b} + \sqrt{b+c} +\sqrt{c+a} \leq \sqrt{6}$
b, $\sqrt{a+1} + \sqrt{b+1} + \sqrt{c+1}< 3,5$
Còn nhiều bài nữa cơ thực ra thì em cũng có biết chút về cái này tôi Cho em làm một chân nha !
Em chưa học cái này nhưng mong Anh chị chỉ giúp em .
À ! mong mấy anh chị chỉ cho em xem có mấy cách chứng minh bất đẳng thức . Lấy ví dụ cho em một thể.Mấy bài trên thì cũng dễ em vừa làm xong mong anh chị làm cho em xem em làm có đúng không nha . Em cảm ơn > icon6.gificon12.gificon12.gificon12.gificon12.gif

câu 1 :đặt a=1/x;b=1/y;c=1/z.sau khi biến đổi áp dụng bđt cauchy cho từng số vơi 1 số khác thêm bớt.

Câu 2: sử dụng bđt bunhia với 1 cà căn(a+b) như vậy là ra

(Thông cảm vì ko bít sd cái này nên ko thể viets đày đủ)



#324
AnhTran2911

AnhTran2911

    Thượng sĩ

  • Thành viên
  • 230 Bài viết

Thách bạn nào làm được, làm được cho 1 Like :like  

Cho $ \left\{\begin{matrix} a,b,c>0 & \\ a+b+c=1& \end{matrix}\right.$

 

Tìm Min của S=$\frac{a^2+b}{b+c}+\frac{b^2+c}{c+a}+\frac{c^2+a}{a+b}$

S=$\frac{a^2+b}{b+c}+\frac{b^2+c}{c+a}+\frac{c^2+a}{a+b}$

  =$\sum$ $\frac{a^2}{b+c}+ $$\sum$ $\frac{b}{b+c}$=$\sum$ ($\frac{a^2}{b+c}+a)+$$\sum$ $\frac{b}{b+c}$-1

  =$\sum$ $\frac{a}{b+c} +$$\sum$ $\frac{b}{b+c}$ -1( do $\sum$a$ ={1}$)

  =$\sum$ $\frac{a+b}{b+c}-1 \geq 2$ ( Theo AM-GM cho 3 số dương).  KL. min biểu thức bằng 2.


        AQ02

                                 


#325
canletgo

canletgo

    Sĩ quan

  • Thành viên
  • 389 Bài viết

Mọi người làm hộ mk bài này với...cảm ơn nhiều  :)

Cho a, b, c và a + b + c $\leq$ 1

CMR: $\sqrt{a^{2}+\frac{1}{a^{2}}}+\sqrt{b^{2}+\frac{1}{b^{2}}}+\sqrt{c^{2}+\frac{1}{c^{2}}}\geq \sqrt{82}$


Alpha $\alpha$ 


#326
nhanlax134

nhanlax134

    Lính mới

  • Thành viên mới
  • 9 Bài viết

cho em hỏi ngu tí em mới học BĐT nên ko phân biệt dược giữa AM-GM và C-S các anh có thể giải thích ch em được không 


~*~
Số hoàn hảo giống như người hoàn hảo, rất hiếm có.
Và tôi không nằm trong số đó 
Perfect numbers like perfect men are very rar
e.
And I'm not one of them
:ukliam2:  :ukliam2:  :ukliam2:  :ukliam2:


#327
hoicmvsao

hoicmvsao

    Thượng sĩ

  • Thành viên
  • 299 Bài viết

Mọi người làm hộ mk bài này với...cảm ơn nhiều  :)

Cho a, b, c và a + b + c $\leq$ 1

CMR: $\sqrt{a^{2}+\frac{1}{a^{2}}}+\sqrt{b^{2}+\frac{1}{b^{2}}}+\sqrt{c^{2}+\frac{1}{c^{2}}}\geq \sqrt{82}$

(1+81)(a^2+b^2)>=(a+9b)^2 hay 82(a^2+b^2)>=(a+9b)^2 => $\sqrt{a^2 +1/a^2}$>=(a+9b)/$\sqrt{82}$.

Tương tự với 2 cái còn lại  thì cần c/m $\sum x +\sum 9/x >= 82$ thì đến đây dễ rồi



#328
nhuleynguyen

nhuleynguyen

    Hạ sĩ

  • Thành viên
  • 88 Bài viết

Giúp mình bài này với!

$A=(x^2-2014x)^2+4026x^2-8108364x+4054183.$

Với giá trị nào của x thì biểu thức A đạt giá trị nhỏ nhất. Tìm giá trị nhỏ nhất đó.


“Life isn't about waiting for the storm to pass...It's about learning to dance in the rain.”

#329
joowon1102712003

joowon1102712003

    Lính mới

  • Thành viên mới
  • 8 Bài viết
May anh xem thu quyen nhung ki nang giaI toan dac sac bdt di. Hay lam

#330
joowon1102712003

joowon1102712003

    Lính mới

  • Thành viên mới
  • 8 Bài viết
Ammg voi cosi la 1 do..no co hai ten goi

#331
joowon1102712003

joowon1102712003

    Lính mới

  • Thành viên mới
  • 8 Bài viết
May ban oI mk moi gia nhap nen khong quen xin chi dan

#332
TranDung04

TranDung04

    Lính mới

  • Thành viên mới
  • 2 Bài viết

Em 2k4, muốn thi chuyên Toán....Em mới học về AM-GM...... mà không hiểu gì... :| Khó quá...


 


Dung :lol: 


#333
Chickey

Chickey

    Binh nhất

  • Thành viên mới
  • 41 Bài viết

Giúp mình bài này với!

$A=(x^2-2014x)^2+4026x^2-8108364x+4054183.$

Với giá trị nào của x thì biểu thức A đạt giá trị nhỏ nhất. Tìm giá trị nhỏ nhất đó.

Mình nghĩ là giải như vầy("không biết có đúng không"):

Gọi:

$y=(x^{2}-2014x)^{2}+4026x^{2}-8108364x+4054183$

Khi đó

$y=x^{4}-4028x^{3}+4060222x^{2}-8108364x+4054183$

<=>${y}'=4x^{3}-12084x^{2}+8120444x-8108364$

Cho y'=0 tìm được:

x=1 hoặc  x=2013 hoặc x=1007

Thế lần lượt các giá trị vào A. Ta thấy:

x=1 và x=2013 cho giá trị A nhỏ nhất ( tại A=2014);

Vậy MinA=2014 .Khi và chỉ khi x=1; x=2013

Nhờ m.n kiểm tra giùm ạ !!


POLITICS ARE FOR THE MOMENT-AN EQUATION IS FOR ETERNITY

                                                                                   -    Albert Einstein-

 

#334
ThinhThinh123

ThinhThinh123

    Trung sĩ

  • Thành viên
  • 138 Bài viết

Mình nghĩ là giải như vầy("không biết có đúng không"):

Gọi:

$y=(x^{2}-2014x)^{2}+4026x^{2}-8108364x+4054183$

Khi đó

$y=x^{4}-4028x^{3}+4060222x^{2}-8108364x+4054183$

<=>${y}'=4x^{3}-12084x^{2}+8120444x-8108364$

Cho y'=0 tìm được:

x=1 hoặc  x=2013 hoặc x=1007

Thế lần lượt các giá trị vào A. Ta thấy:

x=1 và x=2013 cho giá trị A nhỏ nhất ( tại A=2014);

Vậy MinA=2014 .Khi và chỉ khi x=1; x=2013

Nhờ m.n kiểm tra giùm ạ !!

Đạo hàm hay quá anh ơi!!! :like



#335
Trinh Anh

Trinh Anh

    Binh nhất

  • Thành viên mới
  • 20 Bài viết

kiếm 100k nếu có cách giải bài này bằng thuần Cauchy-Schwarz 45182254_1888415967946402_71151316854269

ai làm đc ko ạ



#336
Trinh Anh

Trinh Anh

    Binh nhất

  • Thành viên mới
  • 20 Bài viết

45182254_1888415967946402_71151316854269

chỉ chứng minh dùng cauchy-schwart , ko dùng bất khác  thì có ai làm đc ko ạ



#337
PDF

PDF

    Trung sĩ

  • Thành viên
  • 197 Bài viết

Mình cũng có bài gần tương tự với bài VMO 1996:

Cho $x,y,z\geq 0$ thỏa mãn: $xy+yz+zx+6xyz=9$ . Chứng minh rằng:

$2(x+y+z)\geq xy+yz+zx+3$



#338
Gammaths11

Gammaths11

    Hạ sĩ

  • Thành viên
  • 53 Bài viết

https://diendantoanh...-xyzgeq-xyyzzx/

đây là bài tương tự



#339
ntgeoinq

ntgeoinq

    Lính mới

  • Thành viên mới
  • 1 Bài viết

+,Kết hợp BĐT AM-GM và BĐT phụ-Kĩ thuật đánh giá mẫu
Như đã nói,BĐT phụ là 1 phần cao cả của BĐT,ta sẽ xem nó có ứng dụng rộng rãi như thế nào trong BĐT AM-GM nhé
VD1:(USA MO 1998):Chứng minh với mọi a,b,c thực dương
$\frac{1}{a^3+b^3+abc}+\frac{1}{b^3+c^3+abc}+\frac{1}{c^3+a^3+abc}\leq \frac{1}{abc}$
Giải: BĐT phụ cần áp dụng là $a^3+b^3\geq ab(a+b)$
Áp dụng BĐT trên vào bài toán,ta dễ có đáp án (xin nhường cho các bạn icon6.gif )
*Chú ý:tư tưởng của cách làm trên là đưa về đồng bậc
và đây là 1 VD tương tự ---
(IMO Shortlist 1996) Chứng minh với mọi x,y,z dương có tích bằng 1
$\frac{xy}{x^5+y^5+xy}+\frac{yz}{y^5+z^5+yz}+\frac{zx}{z^5+x^5+zx}\leq 1$
VD2 :(APMO 1998) Chứng minh với mọi a,b,c dương
$(1+\frac{x}{y})(1+\frac{y}{z})(1+\frac{z}{x})\geq 2+\frac{2(x+y+z)}{\sqrt[3]{xyz}}$(*)
Ta có thể coi BĐT $\frac{x}{y}+\frac{y}{z}+\frac{z}{x}\geq \frac{x+y+z}{\sqrt[3]{xyz}}$ (**)là BĐT phụ của BĐT trên không nhỉ ?
Dĩ nhiên BĐT(**) được suy ra trực tiếp từ BĐT(*)
Chứng minh BĐT (*) được suy ra trực tiếp từ đánh giá sau
$\frac{x}{y}+\frac{x}{y}+\frac{y}{z}\geq 3\sqrt[3]{\frac{x^2}{yz}}= \frac{3x}{\sqrt[3]{xyz}}$
VD3:(OLIMPIC 30-4)Chứng minh với mọi a,b,c dương có tổng bình phương bằng 1,ta có BĐT
$\frac{a}{b^2+c^2}+\frac{b}{c^2+a^2}+\frac{c}{a^2+b^2}\geq \frac{3\sqrt{3}}{2}$(1)
Và BĐT phụ của BĐT trên là
với mọi a dương ,BĐT sau đúng$a(1-a^2)\leq \frac{2}{3\sqrt{3}}$ (2)
Ta sẽ chứng minh BĐT (2)
$BDT\Leftrightarrow a^2(1-a^2)^2\leq \frac{4}{27}$
Áp dụng AM-GM ,ta có
$a^2(1-a^2)^2=\frac{1}{2}.2a^2.(1-a^2)(1-a^2)\leq \frac{1}{2}\left [ \frac{2a^2+(1-a^2)+(2-a^2)}{3} \right ]^3=\frac{4}{27}$
BĐT (1) đã được chứng minh
Sau đây là lời giải trọn vẹn
Vì $a^2+b^2+c^2=1$ nên ta viết BĐT thành$\frac{a}{1-a^2}+\frac{b}{1-b^2}+\frac{c}{1-c^2}\geq \frac{3\sqrt{3}}{2}(a^2+b^2+c^2)$
Do đó,ta chỉ cần chứng minh $\frac{a}{1-a^2}\geq \frac{3\sqrt{3}}{2}a^2$ hay $\frac{1}{a(1-a^2)}\geq \frac{3\sqrt{3}}{2}$
Nhưng BĐT đó đã được giải quyết triệt để nav.gif
Hơn nữa,ta còn có BĐT sau (xin dành cho các bạn)
Chứng mnih với mọi a,b,c,d ,e, dương có $a^4+b^4+c^4+d^4+e^4=1$
$\frac{a^3}{b^4+c^4+d^4+e^4}+\frac{b^3}{a^4+c^4+d^4+e^4}+\frac{c^3}{d^4+e^4+a^4+b^4}+\frac{d^3}{e^4+a^4+b^4+c^4}+\frac{e^3}{a^4+b^4+c^4+d^4}\geq \frac{5\sqrt[4]{5}}{4}$
Đây là 1 phần rộng và đòi hỏi lượng kiến thức nhất định,sau đây mình xin đưa ra 1 list các BĐT phụ thường được dùng
1,$(a+b)(b+c)(c+a)\geq \frac{8}{9}(a+b+c)(ab+bc+ca)$
2,$ab+bc+ca\geq \sqrt{3abc(a+b+c)}$
3,$\frac{1}{(1+x)^2}+\frac{1}{(1+y)^2}\geq \frac{1}{1+xy}$
4,$\frac{ab}{c}+\frac{bc}{a}+\frac{ca}{b}\geq a+b+c$
5,$\frac{\sqrt{2a-1}}{a}\leq 1$
6,$abc\geq (a+b-c)(b+c-a)(c+a-b)$
7,$\frac{a^2}{b^2}+\frac{b^2}{c^2}+\frac{c^2}{a^2}\geq \frac{a}{b}+\frac{b}{c}+\frac{c}{a}$
8,$a^3+b^3\geq ab(a+b)$
9,(C-S 2 số)$\sqrt{(a+c)(b+d)}\geq \sqrt{ac}+\sqrt{bd}$
10,Schur: $a^3+b^3+c^3+3abc\geq a^2(b+c)+b^2(c+a)+c^2(a+b)$
*Kĩ thuật đánh giá mẫu
+,Khử căn:Các BĐT chứa căn thứa ở mẫu(có thể cả tử) luôn làm ta khổ sở vì khó xác định được cahs giải,và tư tưởng của ta là làm mất căn thức,sau đây,ta sẽ đến với 1 số VD sau
a,Nhân 1 lượng vừa đủ vào mẫu
Xét VD : Vói mọi a,b,c không âm thỏa mãn $a^2+b^2+c^2=2(ab+bc+ca)$,ta có
$\sqrt{\frac{ab}{a^2+b^2}}+\sqrt{\frac{bc}{c^2+c^2}}+\sqrt{\frac{ca}{c^2+a^2}}\geq \frac{1}{\sqrt{2}}$
Chiếu theo tư tưởng trên ta phải làm mất căn ở mẫu trên,nhưng bằng cách nào?Bình phương lên?đó không phải là lựa chọn tốt vì khối lượng tính toán sẽ không nhỏ?
Xét phân thức $\sqrt{\frac{ab}{a^2+b^2}}$
Ta sẽ nhân thêm$\sqrt{a^2+b^2}$ vào cả tử và mẫu
$\sqrt{\frac{ab}{a^2+b^2}}= \frac{\sqrt{ab(a^2+b^2)}}{a^2+b^2}$
Vẫn ở tư tưởng khử căn,chú ý đến BĐT quen thuộc,ta có $\sqrt{ab(a^2+b^2)}\geq \sqrt{2}ab$
Vậy,t phải chứng minh $\frac{\sqrt{2}ab}{a^2+b^2}+\frac{\sqrt{2}bc}{b^2+c^2}+\frac{\sqrt{2}ca}{c^2+a^2}\geq \frac{1}{\sqrt{2}}$
Hay $\frac{ab}{a^2+b^2}+\frac{bc}{b^2+c^2}+\frac{ca}{c^2+a^2}\geq \frac{1}{2}$( với $a^2+b^2+c^2=2(ab+bc+ca)$
(các bạn chứng minh hộ mình nhé icon6.gif )
b,Nhân 1 lượng vừa đủ vào tử
Cũng chung ý tưởng như ở mục a,tuy nhiên,ở đây ta lại làm mất căn ở tủe số,và sau đó là đánh giá mẫu số bằng AM-GM
VD:Cho các số thực dương a,b,c có tổng bằng 3,chứng minh
$\sqrt{\frac{a^2+b^2+c}{a+b+c^2}}+\sqrt{\frac{b^2+c^2+a}{c+b+a^2}}+\sqrt{\frac{c^2+a^2+b}{c+a+b^2}}\geq 3$
Giải:Ta sẽ làm mất căn ở tử số bởi (bằng cách nhân lượng hợp lý-giống ở mục a)2 lí do sau
a,Sau khi nhân vào tử và mẫu,ta có thể đánh giá mẫu mà không làm đổi chiều bất đẳng thức
b,Xem dlời giải bạn sẽ hiểu
Theo AM-GM ta có $\sqrt{\frac{a^2+b^2+c}{a+b+c^2}}= \frac{a^2+b^2+c}{\sqrt{(a^2+b^2+c)(a+b+c^2)}}\geq \frac{2(a^2+b^2+c)}{a^2+b^2+c^2+a+b+c}$
Các bạn đẫ hiểu tại sao ta làm mất căn ở tử rồi đúng không?Sau khi dùng AM-GM,mẫu thức đã"xem như được đưa về giống nhau)
Xây dựng các BĐT tương tự,ta chỉ cần chứng minh
$\frac{4(a^2+b^2+c^2)+2)a+b+c)}{a^2+b^2+c^2+a+b+c}\geq 3$
hay $a^2+b^2+c^2\geq a+b+c$
với $a+b+c=3$ thì BĐT này không khó chứng mnih
c,Đặt ẩn phụ
Vì kĩ thuật đặt ăn phụ mình đã trình bày nên xin chỉ nêu 1 VD nho nhỏ sau
Chứng mnih với mọi a,b,c dương,ta có
$\frac{1}{a\sqrt{a+b}}+\frac{1}{b\sqrt{b+c}}+\frac{1}{c\sqrt{c+a}}\geq \frac{3}{\sqrt{2abc}}$
Giải BĐT cần chứng minh tương đương $\sqrt{\frac{2bc}{a(a+b)}}+\sqrt{\frac{2ca}{b(b+c)}}+\sqrt{\frac{2ab}{c(c+a)}}\geq 3$
Ta sẽ đặt $\sqrt{\frac{2bc}{a(a+b)}}=x;\sqrt{\frac{2ca}{b(b+c)}}=y;\sqrt{\frac{2ab}{c(c+a)}}=z$
Chú ý thêm rằng xy=$\frac{2c}{\sqrt{(a+b)(b+c)}}$,ta sẽ chứng minh $xy+yz+zx\geq 3$
Tiếp tục đặt $\sqrt{a+b}=p,\sqrt{b+c}=q,\sqrt{c+a}=r$,khi đó,dề thấy $xy=\frac{p^2+q^2-r^2}{qr}$
BĐT cần chứng minh sẽ là $(p^3+q^3+r^3)(p^2q+q^2r+r^2p)\geq q^2p+r^2q+p^2r+3pqr$(Đây là 1 BĐT khá hay,xin nhường cho các bạn)
d,Đánh giá trực tiếp bằng AM-GM
Mình cũng xin chỉ đưa ra 1 VD sau
Chứng minh với mọi số dương a,b,c có tổng bằng 3
$\sqrt{\frac{a+b}{c+ab}}+\sqrt{\frac{b+c}{a+bc}}+\sqrt{\frac{c+a}{b+ca}}\geq 3$
Giải: Áp dụng trực tiếp AM-GM 3 số,ta cần chứng minh
$(a+b)(b+c)(c+a)\geq (c+ab)(a+bc)(b+ca)$
Chú ý rằng theo AM-GM thì $(c+ab)(a+bc)\leq \left [ \frac{c+ab+a+bc}{2} \right ]^2=\frac{(b+1)^2(c+a)^2}{4}$
Xây dựng các BĐT tương tự,ta cần CM $(a+1)(b+1)(c+1)\leq 8$
tuy nhiên ,với a+b+c=3,BĐT trên hiển nhiên đúng
(Phần này sẽ được nới rõ ở kĩ thuật ghép đối xứng)
e,Khử mẫu gián tiếp bằng AM-GM để đưa về cùng mẫu
VD:chứng minh với mọi số thực dương a,b,c có $ab+bc+ca\geq 1$
$\frac{1}{\sqrt{a^2+ab+b^2}}+\frac{1}{\sqrt{b^2+bc+c^2}}+\frac{1}{\sqrt{c^2+ca+a^2}}\geq \frac{9}{(a+b+c)^2}$
Giải: Ta chú ý đến đánh giá sau
$(ab+bc+ca)(a^2+ab+b^2)\leq \frac{(a+b)^2(a+b+c)^2}{4}$
nhưng do $ab+bc+ca\geq 1\Rightarrow \sqrt{a^2+ab+b^2}\leq \frac{(a+b)(a+b+c)}{2}$ hay
$\frac{1}{\sqrt{a^2+ab+b^2}}\geq \frac{2}{(a+b)(a+b+c)}$
phần còn lại dc giải khá đơn giản,xin mời các bạn image019.gif
+,Làm mất mẫu
Tuy rằng các cách đã nêu trên khá "ngon lành" đối với các BĐT chứa căn thức ở mẫu nhưng xem ra lại khá trơ đối với các BĐT không căn thức ở mẫu.Trong trường hợp này,1 trong những cách có thể chọn là khử mẫu.Tư tưởng này đã dc nhắc đến ở phần mở đầu và có phần hơi động chạm tới cân bằng hệ số,tuy nhiên đây là 1 kĩ thuật buộc phải nắm vững
ta xét VD sau (IMO Shorlít 1998):Chứng minh với mọi x,y,z dương có tích bằng 1,ta có BĐT sau
A= $\frac{x^3}{(1+y)(1+z)}+\frac{y^3}{(1+z)(1+x)}+\frac{z^3}{(1+x)(1+y)}\geq \frac{3}{4}$
Giải: Ta sẽ làm gì với BĐT trên,nếu xử lý theo các phương pháp nêu trên thì xem chừng không có hiệu quả
Đánh gíá trung bình nhân-trung bình cộng là điều không thể thực hiện
Đặt ẩn phụ cũng sẽ rất khó khăn wacko.gif
Ta phải tìm cách đánh giá làm sao để mất mẫu đi
Theo cách nghĩ như trên ,ta xét riêg với phân thức $\frac{x^3}{(1+y)(1+z)}$ (các phân thức khác sẽ đánh giá tương tự )
+,Trước tiên,ta kiểm tra (và thấy)BĐT trở thành Đẳng thức khi x=y=z
+,Việc của chúng ta là phải xem ta sẽ áp dụng AM-GM với bộ bao nhiêu số
Để ý điều kiện đề bài là tích 3 số (ở bậc 1),trong khi đó,số mũ của tử số là mũ 3,vì vậy,trong tư tưởng của ta,việc đánh giá phải làm mất được mẫu và đưa tử về bậc 1 (hoặc lớn hơn)-Tuy nhiên,chẳng dại gì mà ta lại đánh giá để đưa về bậc lớn,vì càng lớn thì việc khử "phần thêm" sẽ càng khó,Vậy,ta sẽ áp dụng cho 3 số(Sau này,trong hầu hết các bài toán đánh giá mẫu,thường thì bậc của tử là bậc bao nhiêu thì sẽ đánh giá cho từng ấy số hạng )
+,Đánh giá với những số hạng nào?(cái này mới quan trọng nè)
Ta đã có 1(trên 3)số hạng và phải tìm 2 số hạng còn lại .Tuy nhiên,việc tìm 2 số hạng này không khó(trên tư tưởng làm mất mẫu),2 số hạng cần tìm đương nhiên sẽ có dạng $\frac{1+y}{a},\frac{1+z}{b}$
+,Chỉ cần cho $x=y=z$ là ta tìm được a,b
+,Kết hợp với đề bài là ta có cái ta muốn
+,Sau đây là lời giải đầy đủ
Áp dụng BĐT AM-GM cho 3 số,ta có
$\frac{x^3}{(1+y)(1+z)}+\frac{1+y}{8}+\frac{1+z}{8}\geq \frac{3x}{4}$
Xây dựng các BĐT tương tự ,ta có $A\geq \frac{x+y+z}{2}-\frac{3}{4}$
Bây giờ,chỉ cần chú ý là với diều kiện xyz=1 thì $x+y+z\geq 3\sqrt[3]{xyz}=3$
BĐT xem như đã được giải quyết trọn vẹn
*Tuy nhiên,không dừng lại ở đây,ta còn có thẻ(theo những suy luận không phải không có căn cứ) cố gắng tìm 1 lời giải khác,sẽ có 2 hướng cơ bản sau
+,Điều kiện đề cho xyz=1, liệu ta có hi vọng biến $(x,y,z)\rightarrow (\frac{a}{b},\frac{b}{c},\frac{c}{a})$ không?(theo cách nghĩ của ẩn phụ)
+,Nhận thấy là mẫu chung của các phân thức là $(1+x)(1+z)(1+y)$ và việc đưa về mẫu chung như trên cũng không quá phức tạp,vậy liệu làm thế có cho ta lời giải khác không?
(Phần này xin dành cho những người thích tìm tòi -mình nhác quá mad.gif )

 

Em không nghĩ BĐT $(p^3+q^3+r^3)(p^2q+q^2r+r^2p)\geq q^2p+r^2q+p^2r+3pqr$ đúng đâu ạ.
Anh thử trường hợp $p=q=r=0,5$ xem sao.



#340
huhuhuhu

huhuhuhu

    Binh nhì

  • Thành viên mới
  • 14 Bài viết

kiếm 100k nếu có cách giải bài này bằng thuần Cauchy-Schwarz 45182254_1888415967946402_71151316854269

ai làm đc ko ạ

 

Mình không thấy được đề á bạn






1 người đang xem chủ đề

0 thành viên, 1 khách, 0 thành viên ẩn danh